hello can you help me solve this trigonometry question and in the question use pi to answer it

Hello Can You Help Me Solve This Trigonometry Question And In The Question Use Pi To Answer It

Answers

Answer 1

The area A of a portion of a circle is:

[tex]A=\pi\cdot r^2\cdot\frac{\alpha}{2\pi}[/tex]

Where alpha is the angle of the portion of the circle. So, to find the radius, we clear 'r' from the expression above:

[tex]38=\pi\cdot r^2\cdot\frac{1}{2\pi}\cdot\frac{7}{5}\pi[/tex]

We can cancel the pi on the left of 'r' with the one on the right (the one that's dividing):

[tex]38=r^2\cdot\frac{7\pi}{2\cdot5}[/tex]

So, now we clear 'r':

[tex]\frac{38\cdot2\cdot5}{7\cdot\pi}=r^2[/tex][tex]\sqrt[]{\frac{380}{7\pi}}=r[/tex]

So, the answer is:

[tex]r=\sqrt[]{\frac{380}{7\pi}}km[/tex]


Related Questions

y varies directly with x. if y =75 when x =25, find x when y=25

Answers

Answer:

x = 8.33

Explanation:

y varies directly with x if y can be calculated as a constant k times x. So:

y = k*x

If y is equal to 75 and x is equal to 25, we can calculate the value of k as:

[tex]\begin{gathered} 75=k\cdot25 \\ \frac{75}{25}=\frac{k\cdot25}{25} \\ 3=k \end{gathered}[/tex]

Therefore, y = 3*x

So, to find x when y = 25, we need to replace y by 25 and solve for x as follows:

[tex]\begin{gathered} 25=3\cdot x \\ \frac{25}{3}=\frac{3\cdot x}{3} \\ 8.33=x \end{gathered}[/tex]

Therefore, x is equal to 8.33

Solve the following exponential 4e^3x-1 = 5

Answers

Given

[tex]4e^{3x}-1=5[/tex]

Solving, step-by-step

[tex]\begin{gathered} 4e^{3x}-1=5 \\ e^{3x}=6/3 \\ 3x=\ln 1.5 \\ x=\frac{\ln 1.5}{3} \\ \end{gathered}[/tex]

The price of a necklace was increased by 10% to £121.
What was the price before the increase?

Answers

Answer:

the anwser is 110

Step-by-step explanation:

The price before was £108.9 because 10% of £121 is £12.1 so if you subtract that you would get £108.9

In a right triangle one of the acute angles is 1/4 the size of the other acute angle find the measure of the larger angle

Answers

In a right triangle, there are 2 acute angles and 1 right angle.

Let x be the measure of the larger acute angle, using the triangle sum theorem (that states that the sum of the interior angles of a triangle is 180) we have that:

[tex]\begin{gathered} x+\frac{1}{4}x+90=180 \\ \frac{5}{4}x=180-90 \\ \frac{5}{4}x=90 \\ 5x=360 \\ x=\frac{360}{5} \\ x=72 \end{gathered}[/tex]

The larger acute angle measures 72°.

Solve |x+ 4| = -6 im struggling with this problem

Answers

Recall that:

[tex]\begin{gathered} |a|\ge0 \\ \text{for all a}\in\R. \end{gathered}[/tex]

Therefore the given equation has no solutions.

Answer: Option B.

What is 66253/19
please help this must be turned in by tonight

Answers

Answer:

3487

Step-by-step explanation:

Kareem wants to give $11 to help kids who need school supplies. he also wants to buy a pair of shoes for $39. at least, how much money does he need to earn? brainly

Answers

You would do 11+39 which is 50
He would need to earn $50

What is the percent change from 112 to 96

Answers

well, the change is just 112 - 96 = 16.

now if we take 112 as the 100%, what's 16 off of it in percentage?

[tex]\begin{array}{ccll} amount&\%\\ \cline{1-2} 112 & 100\\ 16& x \end{array} \implies \cfrac{112}{16}~~=~~\cfrac{100}{x} \\\\\\ 7=\cfrac{100}{x}\implies 7x=100\implies x=\cfrac{100}{7}\implies x\approx 14.29[/tex]

A. Put the following values in order from least to greatest:
4²,-√4, √4²,-2√4, 6-47, √

Answers

The order of values in ascending order will be -4<-2<2<2.64<4<16 as the definition of ascending order says, "When numbers are arranged in ascending order, they are done so from smallest to largest".

What is ascending order?

When numbers are arranged in ascending order, they are done so from smallest to largest. Climbing down the stairs of numbers starting with their highest value is another way to think of descending. The slide is descended while moving down it. Ascending order, in which the numbers are arranged from lower value to higher value, is the opposite of descending order. A number can be arranged in ascending order by going from smallest to largest from left to right.

Here,

4²=16

-√4=-2

√4²=4

-2√4=-4

6-4=2

√7=2.64

-4<-2<2<2.64<4<16

According to the definition of ascending order, "When numbers are arranged in ascending order, they are done so from smallest to largest," the order of values in ascending order will be -4<-2<2<2.64<4<16.

To know more about ascending order,

https://brainly.com/question/14404086?referrer=searchResults

#SPJ1

A state offers specialty license plates that contain 5 numbers followed by 2 letters. License plates are assigned randomly. All license plates are equally likely. Findthe number of possible license plates that can be issued using this system.O1.188,137,600 possible license platesO 12,500 possible license platesO 67,600,000 possible license plates039,917,124 possible license plates

Answers

Given:

There are 5 numbers followed by 2 letters.

Solution:

To find the answer, we will have a plate number that consists of 5 numbers followed by 2 letters. In that 5 numbers, each place has 10 choices-from 0 to 9. It will be: 10 * 10 * 10 * 10 * 10 = 100,000 .

Now, for the letters, there are 26 letters in the alphabet. We have two places for the letters and each place have 26 choices-from A to Z that will give us: 26 * 26 = 676

We will now multiply the acquired data for both the numbers and letters to arrive at the correct answer.

100,000 * 676 = 67,600,000

ANSWER: 67,600,000 possible license plates

If 9% of a number equals 3, find 36% of that number.

Answers

The answer is 12 because 9x 4 =36 and 3 x4 =12 therefore 12 is your answer

If AC=13x-17, find x. Round your answer to the nearest tenth if necessary AB= 2x+4 BC= 19

Answers

x=3.7

1) Given that AC is the line segment made up by AB + BC We can set this equation:

AC=AB+BC

13x-17=2x+4+19 Combining like terms

13x-2x -17=2x-2x+23

11x -17=23

11x -17+17 = 23 +17

11x = 40 Dividing both sides by 11

x=3.63 Round to the nearest tenth

x=3.7

an airplane travels 600 miles against the wind in 5 hours, and makes the return trip with the same wind in 2 hours. find the rate of the wind.

Answers

Answer:

The speed of wind is 75 miles / hour

Step-by-step explanation:

Let's assume

speed of airplane is x miles /hour

speed of wind is y miles /hour

An airplane travels 600 miles against the wind in 4 hours

against wind means resultant speed will be less

so, speed =x-y

total distance traveled in 4 hour is

and this has to be equal to 600

Divide both sides by 4

makes the return trip with then same wind in 2 hours

with wind means resultant speed will be more

so, speed =x+y

total distance traveled in 2 hour is

and this has to be equal to 600

Divide both sides by 2

So, we get system of equations

now, we can solve it by adding both equations

now, we can solve for x

now, we can  find y by plugging x into any one equation

a lion sprints after a giraffe. the lions speed is 22.4 meters per second (50 mph). how far will the lion get in fifteen seconds

Answers

Answer:

1100 feet, or 335.28 meters

1
Which graph represents the linear function below?

Answers

The graph that represents the given linear function is attached below.

We are given a function. A function connects an input with an output. It is analogous to a machine with an input and an output. And the outcome is somehow connected to the input. The equation is linear in nature. It represents a straight line. The equation is given below.

(y - 4) = (4/3)(x - 2)

A line's general equation in slope-intercept form is y = mx + c. We will convert the given equation into this form.

y - 4 = (4/3)x - 8/3

y = (4/3)x - 8/3 + 4

y = (4/3)x + 4/3

The slope of the given equation is 4/3. The y-intercept for the given equation is 4/3.

To learn more about functions, visit :

https://brainly.com/question/5975436

#SPJ1

The sum of three numbers is 11. Two times the smallest is 1 less than the largest, while the sum of the largest and smallest is 7.Use a linear system in three variables to find the three numbers.

Answers

ANSWER:

The numbers are 2, 4 and 5

STEP-BY-STEP EXPLANATION:

let x be the smallest, y the one that follows and z the largest

we have the following equations

[tex]\begin{gathered} x+y+z=11 \\ z-1=2x \\ x+z=7 \end{gathered}[/tex]

We solve the system of equations as follows

[tex]\begin{gathered} z-1=2x\rightarrow z=2x+1 \\ x+z=7\rightarrow z=7-x \\ \text{ we equate noth equations} \\ 2x+1=7-x \\ 2x+x=7-1 \\ 3x=6 \\ x=\frac{6}{3}=2 \end{gathered}[/tex]

now, for z:

[tex]\begin{gathered} z=7-2 \\ z=5 \end{gathered}[/tex]

for y:

[tex]\begin{gathered} 2+y+5=11 \\ y=11-5-2 \\ y=4 \end{gathered}[/tex]

Theequation ^4 +^3 −−1=0a. Has four unequal complex rootsb. Has two equal real roots and two equal complex rootsc. Has two unequal real roots and two unequal complex rootsd. Has one real root of multiplicity 2 and a second real root of multiplicity 2e. Is not solvable

Answers

Answer:

Alternative C - Has two unequal real roots and two unequal complex roots

Step-by-step explanation:

The roots of this equation are:

(solved using the computer):

[tex]1,\text{ -1, }\frac{1-i\sqrt[]{3}}{2},\text{ }\frac{1+i\sqrt[]{3}}{2}[/tex]

As we can see, we have two unequal real roots and two unequal complex roots.

Alternative C.

Use the drop-down menus to complete each statement to show why 65–√ is between 12 and 18.

Answers

it is proven that 6√5 between 12 and 18, Irrational numbers

What are Irrational numbers?

The group of real numbers known as irrational numbers are those that cannot be written as a fraction of the form p/q, where p and q are integers. Additionally, the decimal expansion of an irrational integer is neither terminating nor recurring.

Irrational numbers are real numbers that cannot be expressed as a straightforward fraction. These cannot be stated as ratios, such as p/q, where p and q are integers, q0. It is a contradiction of rational numbers.

It has been given in question 4 < 5 < 9 and we have to show 6√5 is between 12 and 18.

For box number 1.

4 < 5 < 9 ⇒ √4 < √5 < √9

For the box number 2.

√4 < √5 < √9    ⇒   2 < √5 < 3

For the box number 3.

we multiply the inequality by 6.

6×(2 < √5 < 3)

⇒ 6×2 < 6×√5 < 6×3

So the 6×2 < 6×√5 < 6×3

⇒ 12 < 6√5 < 18

Hence it is proven that 6√5 between 12 and 18.

Learn more about  Irrational numbers, by the following link.

https://brainly.com/question/20400557

#SPJ1

A yearly coffee club subscription costs $65. Avery received an offer for 30 % off the subscription cost. What is the sale price of the subscription?

Answers

Answer:

$45.50

Step-by-step explanation:

We want to find 70% of the original cost.

[tex].70 \times 65 = 45.50[/tex]

Betsie is going to buy 250 envelopes.
Here is some information about the cost of envelopes in two shops.
Value World
Pack of 10 envelopes for £1.99
Buy 2 packs get 1 pack free
Letters2go
Pack of 25 envelopes for £3.19
Betsie wants to buy the envelopes for the best possible price.
Which shop should Betsie buy the 250 envelopes from?
You must show how you get your answer.

Answers

The shop for which Betsie should buy the 250 envelopes from, using proportions, is Value World.

What is a proportion?

A proportion is a fraction of a total amount, and this fraction is used to build relations, using basic arithmetic operations such as multiplication or division, to obtain the desired measures.

In this problem, the final price is obtained as follows:

Divide the price by the number of packs to find the unit rate.Multiply the unit rate by 250.

Hence the final price at Value World is given as follows:

1.99/10 = 0.119.

She will pay for 2/3 of the 250 packs, as she buys 2 and gets one free, hence:

2/3 x 250 x 0.119 = £19.83.

At Letters2go, the final price is obtained as follows:

3.19/25 x 250 = £31.90.

Due to the lower price, she should purchase her envelopes at Value World.

More can be learned about proportions at https://brainly.com/question/24372153

#SPJ1

Trevor asked his mother how old a tree is in my yard his mother says the sum of 10 and 2/3 of the trees age is in years is equal to (10+2/3)a =50, where A is the tree age in years. his equation is not correct. what error did he makea.the variable a should be mulitplied by 10 only and then added to 2/3b.the variable A should be multiplied by 2/3 only and then added to 10c.The variable A should be multiplied by 50, not by sum of 10 and 2/3d.c.The variable A should be multiplied by 2/3 and 50 and set equal to 10

Answers

Dhamby, this is the solution:

Customer account "numbers" for a certain company consist of 4 letters followed by 4 numbers. How many different account numbers are possible if repetitions of letters and digits are allowed?

Answers

45697600 different account numbers are possible if repetitions of letters and digits are allowed.

Calculation:-

The 4 letters can be chosen in 26 ways each and the two numbers in 10 ways each

Number of account numbers possible = 26^4 * 10^2 = 45697600

What is problem-solving?

Problem-solving is the act of defining a problem; figuring out the purpose of the trouble; identifying, prioritizing, and selecting alternatives for an answer; and imposing an answer.

Problem-solving starts with identifying the issue. As an example, a trainer may need to parent out a way to improve a scholar's overall performance on a writing talent test. To do that, the instructor will overview the writing exams looking for regions for improvement.

Learn more about Problem-solving here:-https://brainly.com/question/23945932

#SPJ1


A report card shows that a student earns 21 points on a test with a total of 25 points. What percentage of the total points has the
student earned on the test?

Answers

Answer: The student got an 84%

Step-by-step explanation:

This one is comparatively simple because 25 is a factor of 100,

we just have to multiply each side by 4 so we get 81 out of 100

Cheers!

Rsmbro

help meeeeeeeeeeeeeee pleaseeeeeee

Answers

Answer: 19.6 feet

Step-by-step explanation:

Using the Pythagorean theorem,

[tex]x^2 +(x+6)^2 =48^2\\\\x^2 +x^2 +12x+36=2304\\\\2x^2 +12x-2268=0\\\\x^2 +6x-1134=0\\\\x=\frac{-6 \pm \sqrt{6^2 -4(1)(-1134)}}{2(1)}\\\\x \approx 30.8 \text{ } (x > 0)\\\\\implies x+(x+6) \approx 67.6\\\\\therefore (x+(x+6))-48 \approx 19.6[/tex]

suppose that g(x) = f(x) -7 which statement best compares the graph of g(x) with the graph of f(x)

Answers

It implies that when F(x) shifts 7 units to the left, we get the value equal to G. (x). As a result, the graph of G(x) is the graph of F(x) with 7 units added to the left.

How is the graph of G related to the graph of f?

The graph of g is the reflection of the graph of f. If g(x) = f(-x), then the graph of g is obtained from the graph of f by reflecting about the y-axis.

For instance, the result of multiplying f(x) and g(x) is h(x) = fg (x), or h(x)=f(x)g (x). if f is the function denoted by f(x) = x2 and g is the function denoted by g(x) = x + 3.

Therefore,

The Correct answer is : The two functions G(x) and F are provided to us (x).

We must determine which assertion most accurately contrasts the graphs of G(x) and F(x).

Suppose

F(x) = x [/ tex]

G(x) = x-7

When x = 1 is substituted

So, F(x) = 1

G(x) = 1-7 = -6

It implies that we obtain the value equal to the value of G when F(x) shifts 7 units to the left (x).

The graph of G(x) is therefore the graph of F(x) with 7 units added to the left.

To learn more about functions refer to:

https://brainly.com/question/25638609

#SPJ1

4. Find the value of x.*60°,150°3x°Ox= 20O х = 30Ox= 60

Answers

The sum of the angles of a quadilateral is 360 degree. So the equation for the sum of angles is,

[tex]\begin{gathered} 90^{\circ}+60^{\circ}+150^{\circ}+3x=360^{\circ} \\ 300^{\circ}+3x=360^{\circ} \\ 3x=360^{\circ}-300^{\circ} \\ x=\frac{60^{\circ}}{3} \\ =20^{\circ} \end{gathered}[/tex]

So the value of x is 20.

When solving the following system of equations, which variable would be the easiest to solve for? {x+6y=25{2x+2y=9 A) the x in the first equationB)the y in the second equationC) the y in the first equationD) the x in the second equation

Answers

SOLUTION

From the first equation, we have

[tex]\begin{gathered} x+6y=25 \\ we\text{ can easily solve for x by subtracting 6y from both sides, we have } \\ x+6y-6y=25-6y \\ x=25-6y \end{gathered}[/tex]

So we can easily solve for x in the first equation.

Hence the answer is option A

Triangle UVW, with vertices U(-6,2), V(-4,6), and W(-8,5), is drawn inside arectangle, as shown below.What is the area, in square units, of triangle UVW?

Answers

Okay, here we have this:

Considering the provided vertices, we are going to calculate the requested area, so we obtain the following:

Then we will first calculate the measure of each side and later with Heron's formula we will find the area, then we have:

[tex]\begin{gathered} u=\sqrt{((-4-(-8))^2+(6-5)^2)} \\ u=\sqrt{4^2+1^2} \\ u=\sqrt{17} \end{gathered}[/tex][tex]\begin{gathered} w=\sqrt{(-6-(-4))^2+(2-6)^2} \\ w=\sqrt{2^2+(-4)^2} \\ w=\sqrt{20} \end{gathered}[/tex][tex]\begin{gathered} v=\sqrt{(-6-(-8))^2+(2-5)^2} \\ v=\sqrt{2^2+(-3)^2} \\ v=\sqrt{13} \end{gathered}[/tex]

Then, the area is:

[tex]\begin{gathered} A=\sqrt{\frac{(\sqrt{13}+\sqrt{17}+\sqrt{20})}{2}(\frac{\sqrt{13}+\sqrt{17}+\sqrt{20}}{2}\sqrt{13})(\frac{\sqrt{13}+\sqrt{17}+\sqrt{20}}{2}\sqrt{17})(\frac{\sqrt{13}+\sqrt{17}+\sqrt{20}}{2}\sqrt{20})} \\ =\sqrt{49} \\ =7 \end{gathered}[/tex]

Finally we obtain that the triangle's area is equal to 7 square units.

( 1 - v ) (5v +9 ) = 0Find v

Answers

Finding an unkown value

We know that if XY = 0 then there are two options: X = 0 or Y = 0

Then, in this case:

If ( 1 - v ) (5v +9 ) = 0​ then

( 1 - v ) = 0 or (5v +9) = 0

We find v for both cases:

1 - v₁ = 0

1 = v₁

5v₂ + 9 = 0

5v₂ = -9

v₂ = -9/5

Answer: v₁ = 1, v₂ = -9/5

Work out the area and the perimeter of each semicircle. Give your answer correct to two decimal places (2 d.p.).
diameter = 24mm

Answers

The area of this semi-circle is 38.28 cm²  and this can be determined by using the formula of area of the semi-circle.

Given :

The diameter of the semi-circle is 10 cm.

The area of the circle is given by the formula:

A=[tex]\pi[/tex]r²

where r is the radius of the circle.

So, the area of the circle whose radius is 5cm is given by:

A=[tex]\pi[/tex]×(5)²
A=25[tex]\pi[/tex]

The area of the circle is 25[tex]\pi[/tex] then the area of the semicircle whose radius is 5cm is given by:

A[tex]s[/tex]= 25/2([tex]\pi[/tex])

A[tex]s[/tex]=12.5[tex]\pi[/tex]

Now, substitute the value of [tex]\pi[/tex] in the above equation.

A[tex]s[/tex]=12.5×3.142

A[tex]s[/tex]=39.28cm²

Learn more from:

https://brainly.com/question/22964077
https://brainly.com/question/16274804

Other Questions
What is 3 times 5???? Marty can make four modifications to his car. He chooses to upgrade the tires, fuel, engine, and driver using the expression 4t + 2f + 3e + 1d to calculate the total cost. Which of these values represents the total cost of these upgrades? 1. He would write, but his hours are as busy / AsWhich technique is being used?Simile, Metaphor, Personification, or HyperboleHow do you figure?(write a sentence explaining your answer)2. My words are little jars / For you to take and puTheir shapes are quaint and beautiful, / And they Adu pick one pen from a box,containing one blue pen, 2 redpens and 3 green pens without.looking into the box. What is theprobability of picking(1) Blue Penif Red pen( Green pen(iv) green or blue penRed or green pen If AABC ADEF, which angle corresponds with angle A in the following image?Side16deee3ce67551e9dd974313e76f08f2 webm 10Blank 1: list 6 different types of geometric principles represented in the picture. What are the key features of a typical mosque ? what is 12 times 12? Find the GCF of 24m^4n and 16m^2n. Convert 6 feet per hr to miles per year Match each system of linear equations with the correct number of solutions. A football team played A games last year. They lost B of those games. What was their winning percentage PLEASE HELP ME OUT WITH THIS X^2+10x+25 Find the slope and y-intercept of the line.y = -3,000 + 30x michael is running for president. the proportion of voters who favor michael is 0.8. a simple random sample of 100 voters is taken. (a) what is the expected value of the sampling distribution of p? hey! I need help/answers to 20 questions. Heres the first question Find the volume of a pyramid with a square base, where the side length of the base is 10.9 m and the height of the pyramid is 4.7 m. Round your answer to the nearest tenth of a cubic meter. 60 POINTS!!! HELPFind any asymptotes or holes in the function below. Explain how you arrived at your answer. SHOW YOUR WORK AND THE STEPS YOU TOOK. the difference between 75% of a number and 20% of the same number is 378.9 what is 40% of that number? equation allows the nurse to use theratio and proportion method todetermine how many tablets the patientrequires?O 100 mg/50 mg x 1 tabletO 50 mg/100 mg x 1 tablet50 mg/1 tablet = 100 mg/ x tablets Who was the first person to write American plays in American vernacular?